Mathcenter Forum

Mathcenter Forum (https://www.mathcenter.net/forum/index.php)
-   ปัญหาคณิตศาสตร์ทั่วไป (https://www.mathcenter.net/forum/forumdisplay.php?f=1)
-   -   Sequences Vs Series (https://www.mathcenter.net/forum/showthread.php?t=24575)

share 01 ตุลาคม 2020 10:37

Sequences Vs Series
 

Sequences Vs Series
ช่วยชี้ กระจ่าง ต่างไฉน
ตัวอย่าง มีด้วย ช่วยเข้าใจ
รีบรีบ ขานไข ไวไวเลย


จูกัดเหลียง 01 ตุลาคม 2020 21:55

มี series ตัวนึงค่อนข้างน่าสนใจครับ กำหนดให้ sequence $a_n = \cases{-1 & , n\not =m^{2020};\exists m \cr 2020n^{\frac{1}{2020}}-1 & \text{ถ้า $n$ เป็นกำลัง $2020$ สมบูรณ์}} $ เราจะได้ว่า $$\sum_{n\ge 1}\dfrac{a_n}{n}=\lim_{n\rightarrow\infty}\left(\sum_{1\le k\le n}\dfrac{1}{k}-\log n\right)=\gamma\approx 0.5772$$


tngngoapm 02 ตุลาคม 2020 07:36

มีเอกลักษณ์ นำเสนอ
เหล่าจอมยุทธ อย่าออเออ
เล่นดูเวอร์ ตามกันไป
โปรดร่วมกัน อภิปราย
ช่วยชี้แจ้ง แถลงไข
ไล่ตัวเลข สนิทใจ
ใครต่อใคร เห็นตรงกัน
เอกลักษณ์ มีชื่อเรียก
Fractorial เศษส่วนย่อย
เรียงตัวเลข บวกกันไป
น่าตกใจ ใครนิยาม

$\frac{1}{1!}=\frac{1}{2!}+\frac{2}{3!}+\frac{3}{4!}+\frac{4}{5!}...$

$\frac{1}{2!}=\frac{2}{3!}+\frac{3}{4!}+\frac{4}{5!}+\frac{5}{6!}+...$

$\frac{1}{3!}=\frac{3}{4!}+\frac{4}{5!}+\frac{5}{6!}+\frac{6}{7!}...$
...
...
$\frac{1}{n!}=\frac{n}{(n+1)!}+\frac{(n+1)}{(n+2)!}+\frac{(n+2)}{(n+3)!}+\frac{(n+3)}{(n+4)!}+...$

หรือ

$$\frac{1}{k!}=\sum_{n = k}^{\infty} \frac{n}{(n+1)!}$$

share 02 ตุลาคม 2020 08:52


ขอบคุณ ท่านทั้งสอง
มอบใจปอง นำเสนอแนะ
ทิ้งห่าง นานแฮะแฮะ
เลยแบะแบะ ไปไม่เป็น


จูกัดเหลียง 02 ตุลาคม 2020 10:53

อ้างอิง:

ข้อความเดิมเขียนโดยคุณ tngngoapm (ข้อความที่ 187389)
มีเอกลักษณ์ นำเสนอ
เหล่าจอมยุทธ อย่าออเออ
เล่นดูเวอร์ ตามกันไป
โปรดร่วมกัน อภิปราย
ช่วยชี้แจ้ง แถลงไข
ไล่ตัวเลข สนิทใจ
ใครต่อใคร เห็นตรงกัน
เอกลักษณ์ มีชื่อเรียก
Fractorial เศษส่วนย่อย
เรียงตัวเลข บวกกันไป
น่าตกใจ ใครนิยาม

$\frac{1}{1!}=\frac{1}{2!}+\frac{2}{3!}+\frac{3}{4!}+\frac{4}{5!}...$

$\frac{1}{2!}=\frac{2}{3!}+\frac{3}{4!}+\frac{4}{5!}+\frac{5}{6!}+...$

$\frac{1}{3!}=\frac{3}{4!}+\frac{4}{5!}+\frac{5}{6!}+\frac{6}{7!}...$
...
...
$\frac{1}{n!}=\frac{n}{(n+1)!}+\frac{(n+1)}{(n+2)!}+\frac{(n+2)}{(n+3)!}+\frac{(n+3)}{(n+4)!}+...$

หรือ

$$\frac{1}{k!}=\sum_{n = k}^{\infty} \frac{n}{(n+1)!}$$

ใช้ induction ครับ โดยเราได้ว่า $\dfrac{1}{k!}-\dfrac{k}{(k+1)!}=\dfrac{1}{(k+1)!}$
ดังนั้นเเล้ว $$\dfrac{1}{(k+1)!}=\dfrac{1}{k!}-\dfrac{k}{(k+1)!}=\left(\sum_{k\le n}\dfrac{n}{(n+1)!}\right)-\dfrac{k}{(k+1)!}=\sum_{k+1\le n}\dfrac{n}{(n+1)!}$$

tngngoapm 02 ตุลาคม 2020 12:01

อ้างอิง:

ข้อความเดิมเขียนโดยคุณ จูกัดเหลียง (ข้อความที่ 187393)
ใช้ induction ครับ โดยเราได้ว่า $\dfrac{1}{k!}-\dfrac{k}{(k+1)!}=\dfrac{1}{(k+1)!}$
ดังนั้นเเล้ว $$\dfrac{1}{(k+1)!}=\dfrac{1}{k!}-\dfrac{k}{(k+1)!}=\left(\sum_{k\le n}\dfrac{n}{(n+1)!}\right)-\dfrac{k}{(k+1)!}=\sum_{k+1\le n}\dfrac{n}{(n+1)!}$$

...step up...ลองร่างการพิสูจน์
$$e-1=\frac{1^2}{2!}+\frac{2^2}{3!}+\frac{3^2}{4!}+\frac{4^2}{5!}+\frac{5^2}{6!}+...$$

จูกัดเหลียง 02 ตุลาคม 2020 15:43

อ้างอิง:

ข้อความเดิมเขียนโดยคุณ tngngoapm (ข้อความที่ 187394)
...step up...ลองร่างการพิสูจน์
$$e-1=\frac{1^2}{2!}+\frac{2^2}{3!}+\frac{3^2}{4!}+\frac{4^2}{5!}+\frac{5^2}{6!}+...$$


ไม่แน่ใจเรื่องเลขนะครับ แต่น่าจะเป็นประมาณนี้ ฝั่งขวาคือ $$\sum_{n\ge 1} \dfrac{(n-1)^2}{n!}=\sum \dfrac{n}{(n-1)!}-2\sum \dfrac{1}{(n-1)!}+\sum \dfrac{1}{n!}=2e-2e+(e-1)=e-1$$
เพราะว่า$$ \sum \dfrac{n}{(n-1)!}x^{n-1}=xe^x+e^x$$

share 03 ตุลาคม 2020 00:14


"ลำดับ"นั้น เรียงเลข เป็นระบบ
จากเริ่มต้น จนจบ มิแปรผัน
พจน์ติดกัน ลบกัน ค่าเท่ากัน
จำให้มั่น "ลำดับ เลขคณิต"

พจน์ติดกัน หารกัน เท่ากันหมด
จงรีบจด จำไว้ ในดวงจิต
เรียกว่า"ลำ- ดับเร- ขาคณิต"
เขียนต่อติด กันไป ถึงปลายทาง

หากจำนวน พจน์นั้น มีจำกัด
เรียก"ลำดับ จำกัด" สมดังอ้าง
หากพจน์นั้น เพิ่มไป ไม่เว้นวาง
เรียกอีกอย่าง ว่า"ลำ- ดับอนันต์"

"อนุกรม" ผลบวก ของลำดับ
รวมทุกพจน์ ผลลัพธ์ เอกฉันท์
อันผลรวม ของลำ- ดับอนันต์
เรียก"อนุกรม อนันต์" คู่กันไป

"อนุกรม จำกัด" จำกัดสิทธิ
"อนุกรม เลขคณิต" คิดเองได้
"อนุกรม เรขา คณิต"ไซร้
ชื่อลำดับ นั่นไง ใช้เหมือนเรา

ค่า limit ลำดับ an เท่ากับศูนย์
จะสมบูรณ์ อนุกรม ต้องลู่เข้า
หาก limit เป็นอื่น เกินคาดเดา
อนุกรม นั้นเล่า ลู่ออกเอย.


Marwin
Pantip


tngngoapm 07 ตุลาคม 2020 07:59

อ้างอิง:

ข้อความเดิมเขียนโดยคุณ share (ข้อความที่ 187397)

"ลำดับ"นั้น เรียงเลข เป็นระบบ
จากเริ่มต้น จนจบ มิแปรผัน
พจน์ติดกัน ลบกัน ค่าเท่ากัน
จำให้มั่น "ลำดับ เลขคณิต"

พจน์ติดกัน หารกัน เท่ากันหมด
จงรีบจด จำไว้ ในดวงจิต
เรียกว่า"ลำ- ดับเร- ขาคณิต"
เขียนต่อติด กันไป ถึงปลายทาง

หากจำนวน พจน์นั้น มีจำกัด
เรียก"ลำดับ จำกัด" สมดังอ้าง
หากพจน์นั้น เพิ่มไป ไม่เว้นวาง
เรียกอีกอย่าง ว่า"ลำ- ดับอนันต์"

"อนุกรม" ผลบวก ของลำดับ
รวมทุกพจน์ ผลลัพธ์ เอกฉันท์
อันผลรวม ของลำ- ดับอนันต์
เรียก"อนุกรม อนันต์" คู่กันไป

"อนุกรม จำกัด" จำกัดสิทธิ
"อนุกรม เลขคณิต" คิดเองได้
"อนุกรม เรขา คณิต"ไซร้
ชื่อลำดับ นั่นไง ใช้เหมือนเรา

ค่า limit ลำดับ an เท่ากับศูนย์
จะสมบูรณ์ อนุกรม ต้องลู่เข้า
หาก limit เป็นอื่น เกินคาดเดา
อนุกรม นั้นเล่า ลู่ออกเอย.


Marwin
Pantip


ถ้ามองแยก แตกต่าง
ไม่หลากหลาย จะกลับกลาย
คับแคบ ไม่ประสงค์
ถ้าเคล้ากัน แบ่งปัน
นั้นมั่นคง อย่าทะนง
ไม่ตรง ไปตรงมา

ผลรวม...เลขคณิต-เรขาคณิต...

ผลรวมของความสัมพันธ์ผสมที่อยู่ในรูป...

$$(a_1)[a_1']+(a_1+d)[a_1'r]+(a_1+2d)[a_1'r^2]+...+(a_1+(n-1)d)[a_1'r^{(n-1)}]...โดยที่...|r|<1$$

ผลรวมจะลู่เข้าสู่...$a_1(\frac{a_1'}{1-r})+d(\frac{a_1'r}{(1-r)^2})$


อ้างอิง:

ข้อความเดิมเขียนโดยคุณ จูกัดเหลียง (ข้อความที่ 187396)
ไม่แน่ใจเรื่องเลขนะครับ แต่น่าจะเป็นประมาณนี้ ฝั่งขวาคือ $$\sum_{n\ge 1} \dfrac{(n-1)^2}{n!}=\sum \dfrac{n}{(n-1)!}-2\sum \dfrac{1}{(n-1)!}+\sum \dfrac{1}{n!}=2e-2e+(e-1)=e-1$$
เพราะว่า$$ \sum \dfrac{n}{(n-1)!}x^{n-1}=xe^x+e^x$$

...อีกซักข้อ

$$(\frac{1}{1^2})(\frac{1}{1^2})+(\frac{1}{2^2})(1+\frac{1}{2^2})+(\frac{1}{3^2})( 1+\frac{1}{2^2}+\frac{1}{3^2})+(\frac{1}{4^2})( 1+\frac{1}{2^2}+\frac{1}{3^2}+\frac{1}{4^2})+...
=\frac{7}{360}\pi ^4$$
เครดิต:ออยเลอร์

share 07 ตุลาคม 2020 08:12


ถ้ามองแยก แตกต่าง
ไม่หลากหลาย จะกลับกลาย
คับแคบ ไม่ประสงค์
ถ้าเคล้ากัน แบ่งปัน
นั้นมั่นคง อย่าทะนง
ไม่ตรง ไปตรงมา

tngngoapm


ว้าว ๆๆๆ ยอด ๆๆๆ

จูกัดเหลียง 08 ตุลาคม 2020 00:15

*(L.Euler)*

$$\dfrac{1}{1^2}\left(\dfrac{1}{1^2}\right)+\dfrac{1}{2^2}\cdot\left(\dfrac{1}{1^2}+\dfrac{1}{2^2}\right)+\dfrac{1}{3^2}\cdot\le ft(\dfrac{1}{1^2}+\dfrac{1}{2^2}+\dfrac{1}{3^2}\right)+\dfrac{1}{4^2}\cdot\left(\dfrac{1}{1^2}+\dfrac{1}{2^2}+\dfrac{1}{3^2}+\df rac{1}{4^2} \right)+\dots
=\dfrac{7\pi ^4}{360}$$

ให้ $\mathscr M$ คือก้อนทางซ้าย เนื่องจาก $\displaystyle \zeta(2)=\dfrac{\pi^2}{6}, \zeta(4)=\dfrac{\pi^4}{90}$
$\displaystyle\mathscr M =\zeta(2)^2-\left(\dfrac{1}{2^2}\left(\dfrac{1}{1^2}\right)+\dfrac{1}{3^2}\left(\dfrac{1}{1^2}+\dfrac{1}{2^2}\right)+\dfrac{1}{4^2}\left(\df rac{1}{1^2}+\dfrac{1}{2^2}+\dfrac{1}{3^2}\right)+\dfrac{1}{5^2}\left(\dfrac{1}{1^2}+\dfrac{1}{2^2}+\dfrac{1}{3^2}+\dfrac{1}{4^2} \right)+\dots\right)$
$\displaystyle =\zeta(2)^2-\left(1-\dfrac{1}{1^4}+\dfrac{1}{2^2}\left(\dfrac{1}{1^2}+\dfrac{1}{2^2}\right)-\dfrac{1}{2^4}+\dfrac{1}{3^2}\cdot\left(\dfrac{1}{1^2}+\dfrac{1}{2^2}+\dfrac{1}{3^2}\right)-\dfrac{1}{3^4}+\dfrac{1}{4^2}\cdot\left(\dfrac{1}{1^2}+\dfrac{1}{2^2}+\dfrac{1}{3^2}+\dfrac{1}{4^2}\right)-\dfrac{1}{4^4}+\dots\right)$
$=\zeta(2)^2-\left(\mathscr M-\zeta(4)\right)$

$\therefore \mathscr M=\dfrac{1}{2}\left(\dfrac{\pi^4}{36}+\dfrac{\pi^4}{90}\right)=\dfrac{7\pi^4}{360}$ ตามต้องการ


เวลาที่แสดงทั้งหมด เป็นเวลาที่ประเทศไทย (GMT +7) ขณะนี้เป็นเวลา 13:17

Powered by vBulletin® Copyright ©2000 - 2024, Jelsoft Enterprises Ltd.
Modified by Jetsada Karnpracha